Đến nội dung

Hình ảnh

$\boxed{\textrm{TOPIC}}$ ÔN THI VÀO $\boxed{\textrm{THPT CHUYEN}}$ LỚP 10 CHUYÊN TOÁN


  • Chủ đề bị khóa Chủ đề bị khóa
Chủ đề này có 150 trả lời

#121
Ngoc Hung

Ngoc Hung

    Đại úy

  • Điều hành viên THCS
  • 1547 Bài viết

Đóng góp bài hình nhé:

Cho tam giác đều ABC. Gọi D là điểm đối xứng của B qua đường thẳng AC. Đường thẳng d qua B cắt các đường thẳng AD, CD lần lượt tại M, N. Các đường thẳng AN và CM cắt nhau tại E. Chứng minh rằng bốn điểm A, C, D, E cùng thuộc 1 đường tròn 



#122
vda2000

vda2000

    Sĩ quan

  • Thành viên
  • 301 Bài viết

Câu 5: ( 2 điểm)

Cho a,b,c là độ dài ba cạnh của một tam giác và x,y,z là ba số thực thoả mãn:
$ax+by+cz+a+b+c=0$.Chứng minh rằng:
$xy+yz+zx+2x+2y+2z+3 \leq 0$

Theo nguyên lí Đi-rích-lê, tồn tại ít nhất $2$ trong $3$ số: $x+1;y+1;z+1$ cùng không dương hoặc không âm, giả sử: $(y+1)(z+1)\geq 0$

Ta có: $a(x+1)+b(y+1)+c(z+1)=0$

$\Leftrightarrow a(x+1)=-b(y+1)-c(z+1)$

$\Leftrightarrow x+1=\frac{-b(y+1)-c(z+1)}{a}$

ĐPCM $\Leftrightarrow (x+1)(y+1)+(y+1)(z+1)+(z+1)(x+1)\leq 0$

$\Leftrightarrow (x+1)((y+1)+(z+1))+(y+1)(z+1)\leq 0$

$\Leftrightarrow\frac{[-b(y+1)-c(z+1)][(y+1)+(z+1)]}{a}+(y+1)(z+1)\leq 0$

$\Leftrightarrow -b(y+1)^2-c(z+1)^2-(b+c)(y+1)(z+1)+a(y+1)(z+1)\leq 0$

$\Leftrightarrow -b(y+1)^2-c(z+1)^2-(b+c-a)(y+1)(z+1)\leq 0$

Ta có: $-b(y+1)^2\leq 0$ và $-c(z+1)^2\leq 0$ và $-(b+c-a)(y+1)(z+1)\leq 0$ (BĐT trong tam giác)

$\Leftrightarrow DPCM$


Bài viết đã được chỉnh sửa nội dung bởi vda2000: 13-05-2015 - 21:33

$\boxed{\textrm{Silence is the peak of contempt!}}$

If you see this, you will visit my facebook.....!


#123
vda2000

vda2000

    Sĩ quan

  • Thành viên
  • 301 Bài viết
 

Câu 4 : ( 6,0 điểm) Trên đường tròn C tâm O, bán kính R vẽ dây AB < 2R.Từ A,Bvẽ các tiếp tuyến Ax,By với đường tròn C .Lấy điểm M bất kì trên cung nhỏ AB .Gọi H,K,I lần lượt là chân các đường vuông góc hạ từ M xuống AB,Ax và By.

1) CMR: $MH^2 = MK.MI $
2) Gỉa sử AM cắt KH tại E,BM cắt HI tại F.CMF: EF là tiếp tuyến chong của hai đường tròn ngoại tiếp các tam giác MEK,MFI.
3) Gọi D là giao điểm thứ hai của hai đường tròn ngoại tiếp các tam giác MEK và MFI.CMR: khi M di chuyển trên cung nhỏ AB thì đường thẳng DM luôn đi qua một điểm cố định .

abc.jpg

1) Dựa vào tứ giác nội tiếp và góc của đường tròn, ta suy ra:

$\widehat{MKH}=\widehat{MAH}=\widehat{MBI}=\widehat{MHI}=a$

và: $\widehat{MHK}=\widehat{MAK}=\widehat{MBH}=\widehat{MIH}=b$

Suy ra: $\Delta {MHK}\sim\Delta {MIH}$ ($g,g$)

Do đó, $MH^2=MI.MK$

2) Ta có: $\widehat{KMH}+\widehat{MHI}=\widehat{EHF}=a+b$

Mà $\Delta MAB$ có: $\widehat{MAB}+a+b=180^{\circ}$

Do đó, $\widehat{EMF}+\widehat{EHF}=180^{\circ}$

Nên: Tứ giác $EMFH$ nội tiếp, suy ra:

$\widehat{MEF}=\widehat{MHF}=\widehat{MKE}$

Dễ dàng chỉ ra $EF$ là đường tròn ngoại tiếp $\Delta MEF$, tương tự với $\Delta MFI$ rồi có đpcm

3) Gọi như hình vẽ: Dễ dàng chỉ ra:

$LE^2=LF^2 (=LM.LD)$

$\Rightarrow LE=LF$ nên $L$ là trung điểm $EF$

Có $EF// BC$ ( dựa vào: $\widehat{MEF}=\widehat{MHF}=\widehat{MAB}$)

Nhờ vào định lí Ta lét, chỉ ra $G$ là trung điểm $AB$, là điểm cố định nên có đpcm


Bài viết đã được chỉnh sửa nội dung bởi vda2000: 13-05-2015 - 21:49

$\boxed{\textrm{Silence is the peak of contempt!}}$

If you see this, you will visit my facebook.....!


#124
vda2000

vda2000

    Sĩ quan

  • Thành viên
  • 301 Bài viết

Câu 5:

Dễ thấy $x=0$ không phải là nghiệm chung của 2 phương trình

Gọi nghiệm chung là $x_0$ thì ta có:

$$P(x_0)=Q(x_0)=0$$

$$\Leftrightarrow x_0^4+ax_0^3+bx_0^2+cx_0+1=x_0^4+cx_0^3+bx_0^2+ax_0+1$$

$$\Leftrightarrow (a-c)x_0^3=(a-c)x_0$$

$$\Leftrightarrow x_0^2=1\Leftrightarrow x_0=\pm 1$$

 

Vậy nghiệm chung của 2 phương trình là $\pm 1$

Hình như đề bảo tìm tất cả các nghiệm còn lại của phương trình.

Thay $x=1$ vào PT: $P(x)=0$, ta có: $a+b+c=-2$

         $x=-1$vào PT: $P(x)=0$, ta có: $-a+b-c=-2$

Do đó, $b=-2$; $a=-c$

$P(x)=0\Leftrightarrow x^4+ax^3-2x^2-ax^2+1=0$

$\Leftrightarrow (x^2-1)(x^2+ax-1)=0$

PT có tập nghiệm $S_1={1;-1;\frac{\sqrt{a^2+4}-a}{2};\frac{-\sqrt{a^2+4}-a}{2}}$

Tương tự với $Q(x)=0$


Bài viết đã được chỉnh sửa nội dung bởi vda2000: 13-05-2015 - 21:57

$\boxed{\textrm{Silence is the peak of contempt!}}$

If you see this, you will visit my facebook.....!


#125
Dinh Xuan Hung

Dinh Xuan Hung

    Thành viên nổi bật 2015

  • Thành viên nổi bật 2016
  • 1396 Bài viết

$\boxed{5}$

Câu 1: Với mọi số nguyên dương n. Chứng minh rằng $(3+\sqrt{5})^n+(3-\sqrt{5})^n$ là số nguyên dương.

Câu 2:Cho ba số thực dương $x, y, z$ Thỏa mãn : $5x^2 + 4y^2 + 3z^2 + 2xyz = 60$.Tìm giá trị lớn nhất của biểu thức $P = x + y + z$

Câu 3:a/ Giải hệ phương trình $\left\{\begin{matrix} (x^4+1)(y^4+1)=4xy\\ \sqrt[3]{x-1}-\sqrt{y-1}=1-x^3 \end{matrix}\right.$

b/ Tìm các số nguyên x, y sao cho $2x - 2\sqrt{y+2}=2\sqrt{2x+1}-y$

Câu 4:a/ Tìm các nghiệm của phương trình $2x^{2}+4x +3a=0(1)$, biết rằng phương trình (1) có một nghiệm là số đối của một nghiệm nào đó của phương trình $2x^{2}-4x -3a=0$

b/ Cho hệ thức $x^{2}+(x^2 + 2)y+6x+9=0$ với x, y là các số thực. Tìm giá trị nhỏ nhất của y.

Câu 5:Cho đoạn thẳng BC có M là trung điểm . Gọi H là một điểm của đoạn thẳng BM (H khác các điểm B và M). Trên đường thẳng vuông góc với BC tại H lấy điểm A sao cho $\widehat{BAH}=\widehat{MAC}$. Đường tròn tâm A bán kính AB cắt đoạn thẳng BC tại điểm thứ hai ở D và cắt đoạn thẳng AC tại E. Gọi P là giao điểm của AM và EB.

         a/ Đặt AB = r, tính tích DH.AM theo r.

         b/ Gọi $h_{1},h_{2},h_{3}$ lần lượt là khoảng cách từ điểm P đến các đường thẳng BC, Ca, AB. Chứng minh rằng $\frac{h_{2}}{AB}+\frac{h_{3}}{AC}<1-\frac{2h_{1}}{BC}$

         c/ Gọi Q là giao điểm thứ hai của hai đường tròn ngoại tiếp hai tam giác APE và BPM. Chứng minh rằng tứ giác BCEQ là tứ giác nội tiếp.

Câu 6: Cho hai số nguyên $x$,$y$. Chứng minh rằng $(x-y)(x-2y)(x-3y)(x-4y)+y^{4}+2$ không thể là một số chính phương
Câu 7: Cho các số thực $a$,$b$,$c$ thoả mãn $a\geq 0$,$b\geq 0$, $c\geq 1$ và $a+b+c=2$.Tìm GTLN của $T=(6-a^{2}-b^{2}-c^{2})(2-abc)$
Câu 8:Cho đường tròn ($O$) đường kính $BC$. Trên tia đối của tia $BC$ lấy điểm $A$ khác $B$. Kẻ các tiếp tuyến $AD$,$AE$ của ($O$) ($D$,$E$ là các tiếp điểm). Kẻ $DH$ vuông góc với $EC$ tại $H$. Gọi $K$ là trung điểm của $DH$, $I$ là giao điểm của $AC$ và $DE$. $CK$ cắt ($O$) tại $Q$ khác $C$, $AQ$ cắt ($O$) tại $M$ khác $Q$. Chứng minh rằng

a. $AB.CI=AC.BI$

b. $QD$ vuông góc với $QI$

c. $DM$ song song với $OC$

Câu 9: Trong mặt phẳng cho $7$ điểm (không có $3$ điểm nào thẳng hàng). Gọi $h$ là độ dài lớn nhất trong các đoạn thẳng nối $2$ trong $7$ điểm đã cho. Chứng minh rằng tồn tại ít nhất $1$ tam giác có các đỉnh là $3$ trong số $7$ điểm đã cho thoả mãn diện tích nhỏ hơn $\frac{h^{2}(4\pi -3\sqrt{3})}{24}$

Câu 10:Cho ba số thực dương $a,b,c$ thỏa mãn $a+b+c=6$.Tìm giá trị nhỏ nhất của biểu thức:$\frac{\sqrt{a^2+ab+b^2}}{bc+4}+\frac{\sqrt{b^2+bc+c^2}}{ca+4}+\frac{\sqrt{c^2+ca+a^2}}{ab+4}$

                                                                                                          HẾT                                                                                                              

Spoiler


Bài viết đã được chỉnh sửa nội dung bởi Dinh Xuan Hung: 15-05-2015 - 20:14


#126
congdaoduy9a

congdaoduy9a

    Sĩ quan

  • Thành viên
  • 338 Bài viết

Câu 6 : $(x-y)(x-2y)(x-3y)(x-4y)+y^{4}+2=(x^{2}-5xy+5y^{2})^{2}+2$ $\equiv 2;3(mod4)$

Vì $a^{2}\equiv 0;1(mod4)$ nên bt đã cho không phải số chính phương



#127
congdaoduy9a

congdaoduy9a

    Sĩ quan

  • Thành viên
  • 338 Bài viết

Câu 3: b) Dễ dàng nhận thấy $\sqrt{y+2};\sqrt{2x+1}$ nguyên vì nếu nó hữu tỉ hay vô tỉ thì vô lí 

PT: $\Leftrightarrow (\sqrt{2x+1}-1)^{2}+(\sqrt{y+2}-1)^{2}=5=1+4$

Đến đây xét các TH là ra

Câu 4: b)Khi hai pt có nghiệm thì theo VIET ta có pt(1) $ x_1+x_2=2$

pt(2) $x_1'+x_2'=-2$

$\Rightarrow x_1+x_2+x_1'+x_2'=0$

Mà pt (1) có 1 nghiệm đối với pt (2) nên giả sử đó là $x_1;x_1'$. 

$\Rightarrow x_2+x_2'=0$

Do đó $x_2;x_2'$ cũng là số đối của nhau.Mà cũng theo VIET 

$x_1x_2=\frac{3}{2}a;x_1'x_2'=\frac{-3}{2}a$

$(-x_1)(-x_2)=\frac{-3}{2}a=x_1x_2=\frac{3}{2}a$

$\Rightarrow a=0$

$x_1=0;x_2=-2$



#128
Dinh Xuan Hung

Dinh Xuan Hung

    Thành viên nổi bật 2015

  • Thành viên nổi bật 2016
  • 1396 Bài viết

$\boxed{5}$

 

Câu 2:Cho ba số thực dương $x, y, z$ Thỏa mãn : $5x^2 + 4y^2 + 3z^2 + 2xyz = 60$.Tìm giá trị lớn nhất của biểu thức $P = x + y + z$

 

                                                                                                          HẾT                                                                                                              

Spoiler

Câu 2:Gỉa thiết suy ra 4$y^{^{2}}$ <60 va 3$z^{2}$ <60 hay $y^{^{2}}$ <15 và $z^{2}$ <20

Ta có : 5$x^{2}$ + 2$x.yz$ + 4$y^{2}$ + 3$z^{2}$ =60 coi là phương trình bậc 2 ẩn là x

       $\Delta$=4$y^{2}z^{2}$-20(4$y^{2}$+3$z^{2}$)

                      =(15-$y^{2}$)(20-$z^{2}$)

   Từ đó $x$ = $\frac{-yz+\sqrt{(15-y^{2})(20-z^{^{2}})}}{5}$ 

                     $\leq$$\frac{-yz+\frac{1}{2}(15-y^{2}+20-z^{2})}{5}$

                     = $\frac{35-(y+z)^{2}}{10}$

     Suy ra $x+y+z\leq \frac{35-(y+z)^{2}+10(y+z)}{10}$

                      =$\frac{60-(y+z-5)^{2}}{10}\leq 6$     

      Dấu bằng xảy ra: $\left\{\begin{matrix}x=1 & & \\ y=2 & & \\ z=3 & & \end{matrix}\right.$

        Vậy max P=6

Spoiler



#129
Dinh Xuan Hung

Dinh Xuan Hung

    Thành viên nổi bật 2015

  • Thành viên nổi bật 2016
  • 1396 Bài viết

$\boxed{5}$

Câu 1: Với mọi số nguyên dương n. Chứng minh rằng $(3+\sqrt{5})^n+(3-\sqrt{5})^n$ là số nguyên dương.

 

                                                                                                          HẾT                                                                                                              

Spoiler

Dùng công thức truy hồi:

Đặt Un=$(3+\sqrt{5})^{n}+(3-\sqrt{5})^{n}$

Đặt $(3+\sqrt{5})^{n} = A$
Đặt $(3+\sqrt{5})^{n} = B$
 $Un=A^{n}+B^{n}$
 Un+1=$A^{n}.(3+\sqrt{5})+B^{n}.(3-\sqrt{5})$
 Un+2=$A^{n}.(3+\sqrt{5})^{2}+B^{n}.(3-\sqrt{5})^{2}$
Un+2=$6A^{n}.(3+\sqrt{5})-4A^{n}+6B^{n}.(3-\sqrt{5})-4B^{n}$
Un+2=6Un+1-4Un suy ra ĐPCM


#130
Dinh Xuan Hung

Dinh Xuan Hung

    Thành viên nổi bật 2015

  • Thành viên nổi bật 2016
  • 1396 Bài viết

$\boxed{5}$

 

Câu 7: Cho các số thực $a$,$b$,$c$ thoả mãn $a\geq 0$,$b\geq 0$, $c\geq 1$ và $a+b+c=2$.Tìm GTLN của $T=(6-a^{2}-b^{2}-c^{2})(2-abc)$
                                                                          HẾT                                                                                                              

Spoiler

$a+b+c=2,c\geq 1 \Rightarrow a+b\leq 1\Rightarrow a,b\leq 1$

Do đó : $(a-1)(b-1)(c-1)\geq 0\Leftrightarrow abc+a+b+c\geq ab+bc+ca+1\Leftrightarrow abc\geq ab+bc+ca-1$

Lại có : $(a+b+c)^{2}=4\Leftrightarrow 4-a^{2}-b^{2}-c^{2}=2ab+2bc+2ca$

Vậy : $T\leq (2+2ab+2bc+2ca)(3-ab-bc-ca)$

Đặt $ab+bc+ca=t$, ta có : $t\leq (2+2t)(3-t)=8-2(t-1)^{2}\leq 8$

dấu bằng xảy ra khi $c=1$ và $1$ trong $2$ số $a$ hoặc $b$ bằng $1$, số còn lại bằng $0$



#131
congdaoduy9a

congdaoduy9a

    Sĩ quan

  • Thành viên
  • 338 Bài viết

$\boxed{5}$

 

b/ Cho hệ thức $x^{2}+(x^2 + 2)y+6x+9=0$ với x, y là các số thực. Tìm giá trị nhỏ nhất của y.

 

$(y+1)x^{2}+6x+2y+9=0$

$\Delta'_x=9-(y+1)(2y+9)$

$\Leftrightarrow (y+1)(2y+9)\leq 0$

$\Leftrightarrow \frac{-9}{2}\leq y\leq-1 $

Do đó y Min -9/2


Bài viết đã được chỉnh sửa nội dung bởi congdaoduy9a: 15-05-2015 - 22:05


#132
Dinh Xuan Hung

Dinh Xuan Hung

    Thành viên nổi bật 2015

  • Thành viên nổi bật 2016
  • 1396 Bài viết

$\boxed{5}$

 

Câu 10:Cho ba số thực dương $a,b,c$ thỏa mãn $a+b+c=6$.Tìm giá trị nhỏ nhất của biểu thức:$\frac{\sqrt{a^2+ab+b^2}}{bc+4}+\frac{\sqrt{b^2+bc+c^2}}{ca+4}+\frac{\sqrt{c^2+ca+a^2}}{ab+4}$

                                                                                                          HẾT                                                                                                              

Spoiler

$\sum \frac{\sqrt{a^2+ab+b^2}}{bc+4}= \frac{\sqrt{\frac{3}{4}(a+b)^2+\frac{1}{4}(a-b)^2}}{bc+4}\geq \sum \frac{\sqrt{3}}{2}\left ( \sum \frac{a+b}{bc+4} \right )$

Đặt $A=\sum \frac{a+b}{bc+4}\Leftrightarrow 4A=\sum \frac{4(a+b)}{bc+4}\geq \sum \frac{4(a+b)}{\frac{(c+b)^2}{4}+4}=\sum \frac{16(a+b)}{(b+c)^2+16}=B$

Đặt $\left\{\begin{matrix} a+b=x(x>0) & & & \\ b+c=y(y>0) & & & \\ c+a=z(z>0) & & & \end{matrix}\right.$

$\Rightarrow x+y+z=2(a+b+c)=12$

Khi đó:$B=\sum \frac{16x}{y^2+16}$

Ta có:$\frac{16x}{y^2+16}=\frac{x(y^2+16)-xy^2}{y^2+16}=x-\frac{xy^2}{y^2+16}\geq x-\frac{xy^2}{8y}=x-\frac{xy}{8}$

CMTT:$\frac{16y}{z^2+16}\geq y-\frac{yz}{8}$

$\frac{16z}{x^2+16}\geq z-\frac{xz}{8}$

$\Rightarrow B\geq x+y+z-\frac{xy+yz+xz}{8}\geq 12-\frac{(x+y+z)^2}{24}=12-\frac{12^2}{24}=6$

$\Rightarrow 4A\geq 6\Leftrightarrow A\geq \frac{3}{2}\Leftrightarrow \sum \frac{\sqrt{a^2+ab+b^2}}{bc+1}\geq \frac{3\sqrt{3}}{4}$


Bài viết đã được chỉnh sửa nội dung bởi Dinh Xuan Hung: 15-05-2015 - 22:18


#133
vda2000

vda2000

    Sĩ quan

  • Thành viên
  • 301 Bài viết

 

Dùng công thức truy hồi:

Đặt Un=$(3+\sqrt{5})^{n}+(3-\sqrt{5})^{n}$

Đặt $(3+\sqrt{5})^{n} = A$
Đặt $(3+\sqrt{5})^{n} = B$
 $Un=A^{n}+B^{n}$
 Un+1=$A^{n}.(3+\sqrt{5})+B^{n}.(3-\sqrt{5})$
 Un+2=$A^{n}.(3+\sqrt{5})^{2}+B^{n}.(3-\sqrt{5})^{2}$
Un+2=$6A^{n}.(3+\sqrt{5})-4A^{n}+6B^{n}.(3-\sqrt{5})-4B^{n}$
Un+2=6Un+1-4Un suy ra ĐPCM

 

Làm thế này được không nhỉ?

Theo khai triển Newton, ta có:

$(3+\sqrt{5})^n=A+B\sqrt{5}$

$(3-\sqrt{5})^n=A-B\sqrt{5}$ với ($A,B\in\mathbb{Z}$)

Suy ra: $(3+\sqrt{5})^n+(3-\sqrt{5})^n=2A\in\mathbb{Z}$ (đpcm)


$\boxed{\textrm{Silence is the peak of contempt!}}$

If you see this, you will visit my facebook.....!


#134
Dinh Xuan Hung

Dinh Xuan Hung

    Thành viên nổi bật 2015

  • Thành viên nổi bật 2016
  • 1396 Bài viết

$\boxed{5}$

 

Câu 3:a/ Giải hệ phương trình $\left\{\begin{matrix} (x^4+1)(y^4+1)=4xy\\ \sqrt[3]{x-1}-\sqrt{y-1}=1-x^3 \end{matrix}\right.$

 

                                                                                                          HẾT                                                                                                              

Spoiler

Điều kiện $y\geq 1$

$4xy=(x^{4}+1)(y^{4}+1)\geq 4x^{2}y^{2}\Rightarrow 0\leq xy\leq 1$

Mà $y\geq 1\Rightarrow x\leq 1$

Do đó $\sqrt[3]{x-1}-\sqrt{y-1}\leq 0\leq 1-x^{3}$

Vậy hệ đã cho có nghiệm duy nhất $\boxed {(x,y)=(1,1)}$



#135
Dinh Xuan Hung

Dinh Xuan Hung

    Thành viên nổi bật 2015

  • Thành viên nổi bật 2016
  • 1396 Bài viết

$\boxed{5}$

 

Câu 5:Cho đoạn thẳng BC có M là trung điểm . Gọi H là một điểm của đoạn thẳng BM (H khác các điểm B và M). Trên đường thẳng vuông góc với BC tại H lấy điểm A sao cho $\widehat{BAH}=\widehat{MAC}$. Đường tròn tâm A bán kính AB cắt đoạn thẳng BC tại điểm thứ hai ở D và cắt đoạn thẳng AC tại E. Gọi P là giao điểm của AM và EB.

         a/ Đặt AB = r, tính tích DH.AM theo r.

         b/ Gọi $h_{1},h_{2},h_{3}$ lần lượt là khoảng cách từ điểm P đến các đường thẳng BC, Ca, AB. Chứng minh rằng $\frac{h_{2}}{AB}+\frac{h_{3}}{AC}<1-\frac{2h_{1}}{BC}$

         c/ Gọi Q là giao điểm thứ hai của hai đường tròn ngoại tiếp hai tam giác APE và BPM. Chứng minh rằng tứ giác BCEQ là tứ giác nội tiếp.

                                                                                                          HẾT                                                                                                              

Spoiler

a) Ta phát biểu 1 bổ đề quen thuộc:

Cho $\triangle ABC$ nội tiếp $(O)$ và có $AD$ là đường cao. Khi đó, ta luôn có $\widehat{BAH}=\widehat{CAO}$.

======================================

Áp dụng vào bài toán. Ta có ngay $\widehat{BAH}=\widehat{CAO}$ với $O$ là tâm đường tròn ngoại tiếp $\triangle ABC$.

Suy ra $\widehat{CAM}=\widehat{CAO} \quad (1)$.

Mặt khác, ta có $\widehat{ABC}<90^o,\widehat{ACB}<90^o$ nên $O$ luôn nằm trong góc $BAC$.

$M \in [BC]$ nên $O,M$ cùng phía với $AC$.

Từ (1), ta có $AM$ đi qua $O$. Mà lại có $O$ nằm trên trung trực $BC$ nên $M \equiv O$ hay $\widehat{BAC}=90^o$.

Dễ thấy $BH=HD, AM=MB \Rightarrow DH.AM=BH.BM=\dfrac{1}{2}BH.BC=\dfrac{1}{2}BA^2=\dfrac{r^2}{2}$

b)

\[
\begin{array}{l}
 \frac{{h_2 }}{{AB}} + \frac{{h_3 }}{{AC}} < 1 - \frac{{2h_1 }}{{BC}} \\
  \Leftrightarrow \frac{{2S_{PAC} }}{{AB.AC}} + \frac{{2S_{PAB} }}{{AB.AC}} < 1 - \frac{{2h_1 }}{{BC}} \\
  \Leftrightarrow \frac{{2\left( {S_{ABC}  - S_{PBC} } \right)}}{{AH.BC}} < 1 - \frac{{2h_1 }}{{BC}} \\
  \Leftrightarrow 1 - \frac{{h_1 }}{{AH}} < 1 - \frac{{2h_1 }}{{BC}} \\
  \Leftrightarrow \frac{1}{{AH}} > \frac{2}{{BC}} \\
 \end{array}
\]
BĐT cuối luôn đúng vì\[
H \in \left( {BM} \right) \Rightarrow AH < AM = \frac{1}{2}BC \Rightarrow \frac{1}{{AH}} > \frac{2}{{BC}}
\]
Ta có đpcm.

c) $\widehat{QBP}=\widehat{QMP}$ và $\widehat{QAP}=\widehat{QEP}$ nên $\triangle QBE \sim \triangle QMA (g.g)$

Do đó $\frac{QB}{QE}=\frac{QM}{QA} \quad (2)$ và $\widehat{BQE}=\widehat{MQA} \quad (3)$.

Từ (3) ta có $\widehat{BQM}=\widehat{AQE}$. Kết hợp với (2) thì $\triangle QBM \sim \triangle QEA(c.g.c)$

$\Rightarrow \widehat{QBM}=\widehat{QEA} \Rightarrow đpcm$



#136
Dinh Xuan Hung

Dinh Xuan Hung

    Thành viên nổi bật 2015

  • Thành viên nổi bật 2016
  • 1396 Bài viết

$\boxed{5}$

 

Câu 9: Trong mặt phẳng cho $7$ điểm (không có $3$ điểm nào thẳng hàng). Gọi $h$ là độ dài lớn nhất trong các đoạn thẳng nối $2$ trong $7$ điểm đã cho. Chứng minh rằng tồn tại ít nhất $1$ tam giác có các đỉnh là $3$ trong số $7$ điểm đã cho thoả mãn diện tích nhỏ hơn $\frac{h^{2}(4\pi -3\sqrt{3})}{24}$

 

                                                                                                          HẾT                                                                                                              

Spoiler

Giả sử 7 điểm đã cho là A B C D E F và AB=h vẽ $\left ( A;h \right )$ và $\left ( B;h \right )$ cắt nhau tại H và K $\Rightarrow$ giao của 2 đường tròn chứa 7 điểm đã cho Đặt diện tích phần này là S $\Rightarrow \Rightarrow S= \frac{h^{2}\left ( 4\Pi -3\sqrt{3} \right )}{6}$ mà có 4 tam giác rời nhau $\Rightarrow dpcm$



#137
Dinh Xuan Hung

Dinh Xuan Hung

    Thành viên nổi bật 2015

  • Thành viên nổi bật 2016
  • 1396 Bài viết

$\boxed{5}$

 

Câu 8:Cho đường tròn ($O$) đường kính $BC$. Trên tia đối của tia $BC$ lấy điểm $A$ khác $B$. Kẻ các tiếp tuyến $AD$,$AE$ của ($O$) ($D$,$E$ là các tiếp điểm). Kẻ $DH$ vuông góc với $EC$ tại $H$. Gọi $K$ là trung điểm của $DH$, $I$ là giao điểm của $AC$ và $DE$. $CK$ cắt ($O$) tại $Q$ khác $C$, $AQ$ cắt ($O$) tại $M$ khác $Q$. Chứng minh rằng

a. $AB.CI=AC.BI$

b. $QD$ vuông góc với $QI$

c. $DM$ song song với $OC$

 

                                                                                                          HẾT                                                                                                              

Spoiler

 

a, $\text{Đpcm} \Leftrightarrow \dfrac{AB}{AC} = \dfrac{BI}{CI} = \dfrac{BD^2}{DC^2}$

Mặt khác $\triangle ADB \sim \triangle ACD$ nên dễ có điều trên.

b, Để ý có $IK$ là đường trung bình của $\triangle EDH$ nên $IK \perp DH \Rightarrow \angle IKD = 90^\circ$
Mà $\angle DIK = \angle DEC = \angle DQC$ nên $DQIK:tgnt$

$\Rightarrow DQI \ 90^\circ$
c, Kéo dài $QI$ cắt $DO$ tại $L$ thì dễ có $L \in (O)$

Vậy $IQ.IL = IE.ID = ID^2 = IO.IA$

$\Rightarrow AQOL:tgnt$

$\Rightarrow \angle QAI = \angle OLI = \angle QED (= \angle DMQ)$

$\Rightarrow QAEI:tgnt$

Spoiler


Bài viết đã được chỉnh sửa nội dung bởi Dinh Xuan Hung: 15-05-2015 - 22:32


#138
Phanbalong

Phanbalong

    Thượng sĩ

  • Thành viên
  • 216 Bài viết

Đóng góp bài dirichle : 
1) Một kiện tướng cờ vua có 77 ngày để xếp lịch du đấu . Anh ta muốn chơi ít nhất một ván mỗi ngày , nhưng không chơi quá 132 ván . Chứng minh rằng có  1 số ngày liên tục anh  ta đã chơi 21 ván cờ 
2) Có n khách tại 1 bữa tiệc. Chứng minh rằng có 2 khách có cùng số người quen


Bài viết đã được chỉnh sửa nội dung bởi Phanbalong: 17-05-2015 - 21:10

'' Để Đạt Được Thành Tích Bạn Chưa Từng Đạt Được, Bạn Phải Làm Những Việc Mà Bạn Chưa Tứng Làm''


#139
Phanbalong

Phanbalong

    Thượng sĩ

  • Thành viên
  • 216 Bài viết

ta có $P=\frac{1}{x^{2}+xy+y^{2}}+\frac{\frac{1}{9}}{xy}+4xy+\frac{\frac{9}{4}}{9xy}+\frac{23}{36xy}\geq \frac{(1+1/3)^{2}}{1}+2+4\doteq \frac{19}{3}$

Mình thắc mắc bạn chọn điểm rơi như thế nào để ra được kết quả này , mong bạn có thể giải thích giúp


'' Để Đạt Được Thành Tích Bạn Chưa Từng Đạt Được, Bạn Phải Làm Những Việc Mà Bạn Chưa Tứng Làm''


#140
Dinh Xuan Hung

Dinh Xuan Hung

    Thành viên nổi bật 2015

  • Thành viên nổi bật 2016
  • 1396 Bài viết

Mình thắc mắc bạn chọn điểm rơi như thế nào để ra được kết quả này , mong bạn có thể giải thích giúp

Có rất nhiều bạn hỏi mình về kỹ thuật chọn điểm rơi.Nhân tiện hôm nay mình xin giới thiệu cho các bạn TOPIC 

Kỹ thuật chọn điểm rơi trong Bất đẳng thức và cực trị




0 người đang xem chủ đề

0 thành viên, 0 khách, 0 thành viên ẩn danh